Finding Derivatives with a Constant Radius

Click For Summary
To find the rate of change of the surface area A of a right circular cylinder with respect to height h while keeping the radius r constant, the derivative dA/dh can be calculated from the formula A = 2πr(r + h). The user initially distributed the terms but became confused about differentiating h since it appears only once in the equation. The correct approach involves recognizing that the derivative of A with respect to h simplifies to dA/dh = 2πr, as the term involving r remains constant. Clarification is sought on how to express the area in terms of h for differentiation, emphasizing the importance of treating constants appropriately in derivative calculations.
myria36
Messages
2
Reaction score
0

Homework Statement



the total surface area of a right circular cylinder is given by the formula: (A = 2Pir(r + h) ).
where r is the radius and h is the height.
sub part a) find the rate of change of A with respect to h is r remains constant

i know how to take derivatives. the only thing is that in this case, I am not sure how to take the derivative of h since it is only present in one term.

Homework Equations


the derivative equation


The Attempt at a Solution


i first ditributed the 2pir, to yield
2pir^2 + 2pirh
2pir^2(h/h) + 2pirh
h (2pir^2 h^-1 + 2pi r)
now i am stuck here. i can't take the derivative of all the h's in my problem, because one h is still present in the equation.
**below is my attempt to still work with it.
dA/dh = 1 times [-1(2pir^2h^-2) + 2pir
final answer: (-2pir^2h^-2) + 2pir
please can someone guide me on the technique i should use for getting the area to be in terms of h. any and all replies are welcome and appreciated
 
Physics news on Phys.org
Don't ever double post again, ok?
 
Question: A clock's minute hand has length 4 and its hour hand has length 3. What is the distance between the tips at the moment when it is increasing most rapidly?(Putnam Exam Question) Answer: Making assumption that both the hands moves at constant angular velocities, the answer is ## \sqrt{7} .## But don't you think this assumption is somewhat doubtful and wrong?

Similar threads

  • · Replies 1 ·
Replies
1
Views
3K
  • · Replies 4 ·
Replies
4
Views
2K
  • · Replies 6 ·
Replies
6
Views
11K
Replies
11
Views
3K
  • · Replies 5 ·
Replies
5
Views
5K
  • · Replies 9 ·
Replies
9
Views
7K
  • · Replies 5 ·
Replies
5
Views
2K
  • · Replies 9 ·
Replies
9
Views
2K
  • · Replies 2 ·
Replies
2
Views
1K
Replies
9
Views
4K